GMAT Prep Test 1 q16 - Please Help

This topic has expert replies
Newbie | Next Rank: 10 Posts
Posts: 2
Joined: Thu Oct 25, 2007 7:03 pm
Thanked: 1 times

GMAT Prep Test 1 q16 - Please Help

by OneMoreTime » Wed Feb 13, 2008 6:56 pm
I apologize if this exists in another thread but I did search and was unable to find it.

The following problem is really giving me a hard way to go. Can someone please explain it?

The total price of a basic computer and a printer was $2500. If the same printer had been purchased with an enhanced computer whose price was $500 more than the price of the basic computer, then the price of the printer would have been 1/5 of that total. What was the price of the basic computer?

a)$1500
b)$1600
c)$1750
d)$1900
e)$2000

The answer is 'D' but again I'm not sure how this was gotten.

Thanks in Advanced!

Legendary Member
Posts: 789
Joined: Sun May 06, 2007 1:25 am
Location: Southern California, USA
Thanked: 15 times
Followed by:6 members

math translation

by resilient » Wed Feb 13, 2008 7:55 pm
the solution starts out with a = computer and b = printer. this gets our variables straight.

the statement gives the solution of printer and computer = 2500.

so a+b=500. the new computer and printer solution is a+500= ??????? from here I am lost. I am very curious how to solve this out!
Appetite for 700 and I scraped my plate!

User avatar
GMAT Instructor
Posts: 3225
Joined: Tue Jan 08, 2008 2:40 pm
Location: Toronto
Thanked: 1710 times
Followed by:614 members
GMAT Score:800
OneMoreTime wrote:I apologize if this exists in another thread but I did search and was unable to find it.

The following problem is really giving me a hard way to go. Can someone please explain it?

The total price of a basic computer and a printer was $2500. If the same printer had been purchased with an enhanced computer whose price was $500 more than the price of the basic computer, then the price of the printer would have been 1/5 of that total. What was the price of the basic computer?

a)$1500
b)$1600
c)$1750
d)$1900
e)$2000

The answer is 'D' but again I'm not sure how this was gotten.

Thanks in Advanced!
We're told that if we spent an extra $500, the price of the printer would be 1/5 of the total.

We originally spent $2500, so with the extra $500 our new cost is $3000.

We know that the printer costs 1/5(3000) = 600.

Our original cost was $2500, so if we subtract the $600 for the printer, that means that the computer cost us $1900: choose (d).
Image

Stuart Kovinsky | Kaplan GMAT Faculty | Toronto

Kaplan Exclusive: The Official Test Day Experience | Ready to Take a Free Practice Test? | Kaplan/Beat the GMAT Member Discount
BTG100 for $100 off a full course

Junior | Next Rank: 30 Posts
Posts: 16
Joined: Tue Feb 05, 2008 10:47 am
Thanked: 3 times

by davidforsberg » Thu Feb 14, 2008 7:09 am
I did it the long way around and also came up with the correct answer:

C+P=2500 (1st equation)

(C+500)+((C+500+P)/5)=C+500+P (2nd equation)

If you work them out you get P=600 and C=1900
T minus 17 hours

Master | Next Rank: 500 Posts
Posts: 111
Joined: Thu Jan 31, 2008 4:05 pm
Thanked: 18 times
Followed by:1 members

by xilef » Thu Feb 14, 2008 5:36 pm
p+c=2500
p+(c+500)=5p (since printer is 1/5 of the total)

solve for p and c - 600 and 1900

Newbie | Next Rank: 10 Posts
Posts: 2
Joined: Thu Oct 25, 2007 7:03 pm
Thanked: 1 times

RE:

by OneMoreTime » Sat Feb 16, 2008 11:33 am
Thanks all for the help! :D